Find the area using integration.

  • Thread starter Thread starter turbokaz
  • Start date Start date
  • Tags Tags
    Area Integration
Click For Summary
To find the area bounded by the graph f(x)=2x^2, the tangent line at point P(1, f(1)), and the x-axis, the correct approach involves two integrals due to the changing boundaries. The tangent line is identified as y=4x-2, which intersects the curve at (1,2). The area calculation requires integrating the top function minus the bottom function over two separate intervals. The x-intercept of the tangent line must also be considered to accurately determine the limits of integration. The initial attempt of integrating from 0 to 1 is insufficient for capturing the entire area.
turbokaz
Messages
19
Reaction score
0

Homework Statement


Find the area bounded by the graph f(x)=2x^2, point P(1, f(1)), and the x axis.


Homework Equations





The Attempt at a Solution


Point P is a point on a line that is tangent to y=2x^2. So I used this to figure that the line is 4x-2. So now there are two functions, y=2x^2 and y=4x-2, with the two functions intersecting at (1,2). I did the integral from 0 to 1 of top minus bottom, which is 2x^2-4x+2. Taking the antiderivatives, I get 2x^3/3 -2x^2 +2x. From 0 to 1, I get an answer of 2/3. But that isn't correct apparently. The answer choices are 1/8, 1/6, 1/2, 1/3 and 1/4. Where am I wrong?
 
Physics news on Phys.org
turbokaz said:

Homework Statement


Find the area bounded by the graph f(x)=2x^2, point P(1, f(1)), and the x axis.
Your problem description is not as good as it should be. One of the boundaries of the region is the tangent line through (1, f(1)).
turbokaz said:

Homework Equations





The Attempt at a Solution


Point P is a point on a line that is tangent to y=2x^2. So I used this to figure that the line is 4x-2. So now there are two functions, y=2x^2 and y=4x-2, with the two functions intersecting at (1,2). I did the integral from 0 to 1 of top minus bottom, which is 2x^2-4x+2. Taking the antiderivatives, I get 2x^3/3 -2x^2 +2x. From 0 to 1, I get an answer of 2/3. But that isn't correct apparently. The answer choices are 1/8, 1/6, 1/2, 1/3 and 1/4. Where am I wrong?

You aren't taking into full account the tangent line. Over part of the interval, the x-axis is the bottom boundary of the region. Over the other part of the interval, the tangent line is the bottom boundary. You need two integrals, since the integrand is different over the two intervals.

A piece of information that you have neglected is the x-intercept of your tangent line.

BTW, I get one of the values you list as potential answers.
 
Question: A clock's minute hand has length 4 and its hour hand has length 3. What is the distance between the tips at the moment when it is increasing most rapidly?(Putnam Exam Question) Answer: Making assumption that both the hands moves at constant angular velocities, the answer is ## \sqrt{7} .## But don't you think this assumption is somewhat doubtful and wrong?

Similar threads

  • · Replies 4 ·
Replies
4
Views
2K
  • · Replies 3 ·
Replies
3
Views
1K
  • · Replies 6 ·
Replies
6
Views
2K
  • · Replies 10 ·
Replies
10
Views
2K
  • · Replies 20 ·
Replies
20
Views
2K
Replies
1
Views
1K
Replies
5
Views
2K
  • · Replies 6 ·
Replies
6
Views
1K
  • · Replies 9 ·
Replies
9
Views
2K
  • · Replies 105 ·
4
Replies
105
Views
6K